LSAT and Law School Admissions Forum

Get expert LSAT preparation and law school admissions advice from PowerScore Test Preparation.

 Administrator
PowerScore Staff
  • PowerScore Staff
  • Posts: 8917
  • Joined: Feb 02, 2011
|
#33524
Complete Question Explanation

Strengthen—PR. The correct answer choice is (D)

The key to answering this question quickly and efficiently is first to simplify the stimulus, and then to formulate a suitable prephrase. Do not let complex language and unfamiliar terminology throw you off: there is no need to know what a “perihelion” is in order to answer this question.

The author argues that just because some phenomenon is predicted by a scientific theory does not mean that the phenomenon counts as evidence in support of that theory. Why not? Because the phenomenon was well known at the time the theory was developed, so the theory could have been manipulated to account for it.

Note that the question stem is a Strengthen—PR (not a Justify—PR) because of the presence of the word “most” in the question stem, which weakens the force required of the correct answer. In a Strengthen—PR question, the correct answer will provide a premise that, when applied to the specific situation in the stimulus, helps support the conclusion. Since a principle is by definition a broad rule (usually conditional in nature), the presence of the Principle indicator serves to broaden the scope of the question, which requires a more abstract understanding of the underlying relationships in the argument.

In this problem, you must select a principle establishing the conditions under which a particular phenomenon should not count as evidence in favor of a scientific theory:
  • Theory could have been rigged to ..... ..... Phenomenon should NOT count
    account for the phenomenon ..... :arrow: ..... as evidence in support of the theory
Answer choice (D) contains the statement that is closest to this prephrase.

Answer choice (A): The author does not credit Einstein’s theory of relativity with discovering the advance of the perihelion of Mercury. This principle cannot be applied to the facts, as they are described in the stimulus.

Answer choice (B): This principle argues that a phenomenon predicted by a scientific theory should not count as evidence in favor of that theory, unless the theory was developed with that phenomenon in mind. After applying the Unless Equation, this answer choice can be diagrammed as follows:
  • Phenomenon counts as evidence ..... ..... ..... The theory was developed with
    in support of a theory ..... ..... :arrow: ..... ..... that phenomenon in mind
This principle is the exact opposite to the one we are looking for: the theory should not have been developed with the phenomenon in mind.

Answer choice (C): The stimulus does not seek to identify the conditions under which a particular theory could be regarded as “well supported,” and no mention is made regarding the need to account for “all relevant phenomena.”

Answer choice (D): This is the correct answer choice. If Einstein’s theory of relativity was adjusted to account for the perihelion advance, then the perihelion advance should not count as evidence in favor of the theory.

Answer choice (E): The argument makes no attempt to determine if a particular theory should count as predicting some phenomenon. The issue is whether the ability of a theory to predict a phenomenon is sufficient to regard that phenomenon as evidence in support of the theory.

Get the most out of your LSAT Prep Plus subscription.

Analyze and track your performance with our Testing and Analytics Package.